LSAT and Law School Admissions Forum

Get expert LSAT preparation and law school admissions advice from PowerScore Test Preparation.

 Administrator
PowerScore Staff
  • PowerScore Staff
  • Posts: 8919
  • Joined: Feb 02, 2011
|
#23734
Complete Question Explanation

Flaw in the Reasoning. The correct answer choice is (A)

The stimulus observes that over the past 20 years hospital fatalities due to anesthesia decreased through improved training and not through the introduction of safety equipment, and concludes that now that safety equipment actually exists, it will not help cut fatalities.

The argument seems to assume that simply because training caused the improvement without any help from equipment, equipment cannot also help. Abstractly, the argument seems to assume that a sufficient cause is an only cause, and that is what you should remember when you are looking over the choices.

Answer choice (A): This is the correct answer choice. The argument assumes that because Training led to Improvement, Equipment will not lead to Improvement.

Answer choice (B): This choice describes circular reasoning, but you should not select a circular reasoning response when the conclusion proceeds from interpreting evidence, no matter how unconvincing the interpretation. While it is possible that an argument that uses circular reasoning will involve evidence, it must always be clear that the argument begins from a stated principle rather than the evidence, and simply restates the principle as a conclusion.

Answer choice (C): This choice may have seemed attractive, but it renders an actual flaw poorly. An actual flaw is that the argument assumes that the absence of a factor when a result occurred is evidence that the factor will not lead to the result, but this choice erroneously claims that the argument assumes that the absence of Equipment is what Improvement depended on.

Answer choice (D): This choice discusses an internal contradiction, but the stimulus had no such contradiction.

Answer choice (E): This response does not hit on the argument's attempt to eliminate Equipment as a possible cause of Improvement, and focuses unnecessarily on whether Training was actually a cause of Improvement. You should not select a flaw response that does not concern the main line of reasoning.
User avatar
 anureet
  • Posts: 22
  • Joined: Aug 06, 2021
|
#90103
Hello,

Just a quick question. I don't understand why E is wrong. I don't understand what the option is even saying. Is it trying to say that the first event (better training anesthetists) caused a second event (significant safety improvements) and both of them were independent from the third event (increased use of monitoring equipment)?

Regards,
Anureet
User avatar
 atierney
PowerScore Staff
  • PowerScore Staff
  • Posts: 215
  • Joined: Jul 06, 2021
|
#90118
Hello,

Well, in considering whether the answer choice is wrong or right, you must first understand what the answer choice is saying. So, that will be our first consideration. Answer choice is stating that the error in the stimulus' reasoning is that it fails to identify a third cause that actually better explains both the causality of both events at issue. This type of answer choice considers a situation where the argument is claiming that A causes B, when the evidence better supports the fact that a third cause (C) causes both A and B, as opposed the relationship of A causing B as posited.

An example of this would be thinking that your dog's empty food bowl is causing it to bark, when in reality, a stray raccoon has someone entered your house, and has eaten the dog food and is now antagonizing the dog causing it to bark! Here, the stated relationship, empty food bowl causes unhappy dog is better explained by hungry raccoon causes both empty food bowl and unhappy dog (where the dog is obviously unhappy at the raccoon's presence and not purely of the raccoon's thievery!).

Here, the consideration is of improvement in the rate of hospital fatalities (the primary effect), and causes as to this. Thus, the two causes considered both apply equally to the effect, whereas the relationship between them is neither discussed nor relevant. Thus, A is the best answer here.

Let me know if you have further questions on this .
User avatar
 rabanirandhawa
  • Posts: 1
  • Joined: May 28, 2023
|
#101962
Hello,

I just needed some clarification.

"Abstractly, the argument seems to assume that a sufficient cause is an only cause, and that is what you should remember when you are looking over the choices."

Is this not a valid assumption considering the assumptions of LSAT writers is that one and only one cause will cause a certain effect?

Thank you,

Rabani
User avatar
 Jeff Wren
PowerScore Staff
  • PowerScore Staff
  • Posts: 385
  • Joined: Oct 19, 2022
|
#101974
Hi Rabani,

It's important to distinguish the assumptions of the causal arguments on the LSAT from the assumptions of the LSAT writers.

What I mean is that most causal arguments that appear on the LSAT do involve a single stated cause (usually in the argument's conclusion), but it's also important to realize that most causal arguments are inherently flawed. It is extremely difficult to prove for certain that one thing caused another. You need to rule out all other possibilities, rule out coincidence, etc..

For example, a basic causal argument on the LSAT might say "Group A exercised 3 times a week for a year and lost an average of 10 pounds. Therefore, exercise caused them to lose this weight." Here, the person making the argument is basically claiming that exercise was the one and only cause in this situation. While this conclusion may seem pretty reasonable to most people, we don't actually know for certain that exercise did in fact cause the weight loss.

For the purposes of the argument, if we were to introduce a possible alternate cause (such as Group A also went on a low calorie diet), that weakens the claim that the exercise was the one and only cause. Obviously, in the real world, diet and exercise could both contribute to losing weight.

If, in the example above, the argument then concluded that diet could not cause weight loss, that would be the same flaw as appears in this question.

Get the most out of your LSAT Prep Plus subscription.

Analyze and track your performance with our Testing and Analytics Package.